0 Daumen
404 Aufrufe

Aufgabe:

Gibt es eine Möglichkeit 4-1/1^n-1/2^n zu 2(1-1/2^n) umzuwandeln?

4 steht alleine -1/1^n ist ein Bruch und -1/2^n auch.

2 ist alleine (1 ist alleine und -1/2^n) ist ein bruch

Avatar von

Ist das \(4-(\frac{1}{1})^n-(\frac{1}{2})^n \)?

Hallo

1/1=1 also ist es (-1)^n oder -1^n=-1? dasselbe für dein -1/2^n ist das immer negativ oder soll es (-1)^n sein.

wenn es (-1)^n ist dann hast du ja für n ungerade 4-1=3 und für n gerade 5 entsprechend 1/2^n oder -1/2^n , dann kann man 2 so nicht ausklammern.

bei Hochzahlen mit Brüchen musst du deutlich Klammern (oder nicht) setzen

Gruß lul

Apfelmännchen und lul ohne Klammer und n ist im Nenner.


Ich komme leider gar nicht mit den Symbolen hier klar

Apfelmännchen und lul ohne Klammer und n ist im Nenner.


Also so$$4-\frac{1}{1^n}-\frac{1}{2^n} \stackrel{?}{=} 2\left(1-\frac{1}{2^n}\right)$$was aber der identische Term zu dem von Apfelmännchen ist!

Ist das richtig?

Falls ja - probiere \(n=0\)

Ja genau so. Die Sache ist ich versuche eine vollständige Induktion. Wie kann ich da mit links n = 0 auf den rechten Term kommen?

Wie kann ich die linke Seite also 2(1-1/2^n) + das gleiche nur mit n+1 zusammenfassen? Wäre mir große Hilfe, sofern dieser Ansatz korrekt ist...
Wie kann ich da mit links n = 0 auf den rechten Term kommen?

Nun: gar nicht! Wenn Du links \(n=0\) einsetzt, dann kommt heraus:$$4-\frac{1}{1^0}-\frac{1}{2^0}=4-1-1=2$$und rechts$$2\left(1-\frac{1}{2^0}\right) = 2(1-1)=0$$und 2 ist eben nicht gleich 0. Also kann man den einen Term auch nicht in den anderen umwandeln, mit der Voraussetzung, dass \(n \in\mathbb{N}_0\) ist.

Es ist eine vollständige Induktion verlangt mit 2(1-1/2n) rechts der gleichung und summe von k=1 bis n mit 1/2^(k-1) auf der links der gleichung.

Klammer beim rechten Term nur zur Verdeutlichung. In der Aufgabe ist dort keine.



Der rechte Term hat 2n im Nenner.

Kleinste n ist 1

2 Antworten

0 Daumen

Also \( 4-\frac{1}{1^n}-\frac{1}{2^n}=4-1-\frac{1}{2^n}=3-\frac{1}{2^n} \).

Aber \( 2(1-\frac{1}{2^n})=2-\frac{1}{2^{n-1}} =3-1-\frac{1}{2^{n-1}}=3-\frac{2^{n-1}+1}{2^{n-1}}=3-\frac{2^{n}+2}{2^{n}} \).

Avatar von 11 k

Mein Ansatz ist also falsch. Dankesehr.

Wie kann ich die linke Seite also 2(1-1/2^n) + das gleiche nur mit n+1 zusammenfassen? Wäre mir große Hilfe, sofern dieser Ansatz korrekt ist...

Du solltest einmal die Aufgabe und deine Ansätze oben mit angeben. Diese Textbeschreibungen machen ein Verständnis schwierig.

Ich muss die Aufgabe abgeben deshalb meide ich das.

Linke Seite mit Angabe von dieser + das gleiche mit n+1 ist doch verständlich?

Zusammenfassen reicht hier aber nicht, wenn man etwas bestimmtes zeigen soll. Es ist ja unklar, wo du hinwillst.

Auf Mathematikaufgaben gibt es in der Regel kein Urheberrecht, falls du deswegen Bedenken hast.

Ich vermute, dass gar nicht die Gleichheit gezeigt werden soll.

Geht es vielleicht um eine Ungleichungskette?

Es ist eine vollständige Induktion verlangt mit 2(1-1/2^n) rechts der gleichung und summe von k=1 bis n mit 1/2^(k-1) auf der links der gleichung.

Klammer beim rechten Term nur zur Verdeutlichung. In der Aufgabe ist dort keine.


Der rechte Term hat 2^n im Nenner.

Kleinste n ist 1

Freunde ich brauche euch

0 Daumen

Hallo,

... und summe von k=1 bis n mit 1/2^(k-1) auf der links der gleichung.

diese Kleinigkeit hattest Du uns bisher ja verschwiegen!

Es soll also gezeigt werden, dass$$\sum\limits_{k=1}^{n}\frac{1}{2^{k-1}} = 2\left(1-\frac{1}{2^n}\right)$$

Induktionsanfang: \(n=1\) $$\sum\limits_{k=1}^{1}\frac{1}{2^{k-1}} \stackrel{?}{=} 2\left(1-\frac{1}{2^1}\right) \\ \implies \frac{1}{2^0}= 1 = 2\left(1-\frac{1}{2}\right) = 1\space \checkmark$$Übergang von \(n\) auf \(n+1\)$$\begin{aligned} \sum\limits_{k=1}^{n+1}\frac{1}{2^{k-1}} &= \sum\limits_{k=1}^{n}\frac{1}{2^{k-1}} + \frac{1}{2^{n}} \\ &= 2\left(1-\frac{1}{2^n}\right) + \frac{1}{2^{n}} &&|\,\text{nach Vor.} \\ &= 2 - \frac{2}{2^n}+ \frac{1}{2^{n}} \\ &= 2 - \frac{1}{2^{n}} \\&= 2 - \frac{2}{2^{n+1}} \\ &= 2\left(1-\frac{1}{2^{n+1}}\right) \\ &\text{q.e.d.}\end{aligned}$$Gruß Werner

Avatar von 48 k

Ein anderes Problem?

Stell deine Frage

Willkommen bei der Mathelounge! Stell deine Frage einfach und kostenlos

x
Made by a lovely community